Solving used Real Based Methods: $int_0^x frac{t^k}{left(t^n + aright)^m}:dt$












9












$begingroup$


In working on integrals for the past couple of months, I've come across different cases of the following integral:



begin{equation}
Ileft(x,a,k,n,mright) = int_0^x frac{t^k}{left(t^n + aright)^m}:dt
end{equation}



Where $x,ain mathbb{R}^{+}$.



Here the method that I've taken is rather simple and I was curious as to other 'Real' Based methods could be employed with this integral? I also believe that with the conditions I've set on the parameters that it is convergent. If I'm able to expand those conditions, could you please advise.



Interested in special cases too!



The method I took:



First I wanted to bring the 'a' out the front:



begin{equation}
I(x,a,k,n,m) = int_0^x frac{t^k}{left(aleft[left(a^{-frac{1}{n}}tright)^n + 1right]right)^m}:dt = frac{1}{a^m} int_0^x frac{t^k}{left(left(a^{-frac{1}{n}}tright)^n + 1right)^m}:dt
end{equation}

Here let $u = a^{-frac{1}{n}}t$ Thus,



begin{equation}
I(x,a,k,n,m) = frac{1}{a^m} int_0^{a^{-frac{1}{n}}x} frac{left(a^{frac{1}{n}}uright)^k}{left(u^n + 1right)^m}a^{frac{1}{n}}:du = a^{frac{k + 1}{n} - m}int_0^{a^{-frac{1}{n}}x} frac{u^k}{left(u^n + 1right)^m}:du = a^{frac{k + 1}{n} - m}I(a^{-frac{1}{n}}x,1,k,n,m)
end{equation}



From here I will use $I$ in place of $I(x,a,k,n,m)$ for ease of typing. The next step is to make the substitution $w = u^n$ to yield:



begin{equation}
I = a^{frac{k + 1}{n} - m}int_0^{ax^n} frac{w^frac{k}{n}}{left(w + 1right)^m}frac{:dw}{nw^{frac{n - 1}{n}}} = frac{1}{n}a^{frac{k + 1}{n} - m}int_0^{ax^n} frac{w^{frac{k + 1}{n} - 1}}{left(w + 1right)^m}:dw
end{equation}



Here make the substitution $z = frac{1}{1 + w}$ to yield:



begin{align}
I &= frac{1}{n}a^{frac{k + 1}{n} - m}int_1^{frac{1}{1 + ax^n}} z^m left(frac{1 - z}{z}right)^{frac{k + 1}{n} - 1}left(-frac{1}{z^2}right) :dz = frac{1}{n}a^{frac{k + 1}{n} - m}int_{frac{1}{1 + ax^n}}^1 z^{m - frac{k + 1}{n} - 1}left(1 - zright)^{frac{k + 1}{n} - 1}:dz \
&= frac{1}{n}a^{frac{k + 1}{n} - m} left[int_0^1 z^{m - frac{k + 1}{n} - 1}left(1 - zright)^{frac{k + 1}{n} - 1}:dz - int_0^{frac{1}{1 + ax^n}} z^{m - frac{k + 1}{n} - 1}left(1 - zright)^{frac{k + 1}{n} - 1}:dz right] \
&= frac{1}{n}a^{frac{k + 1}{n} - m} left[Bleft(m - frac{k + 1}{n}, frac{k + 1}{n}right) - Bleft( frac{1}{1 + ax^n}; m - frac{k + 1}{n}, frac{k + 1}{n} right)right]
end{align}



Where $B(a,b)$ is the Beta Function and $B(x; a,b)$ is the Incomplete Beta Function.



And so, we arrive at:



begin{equation}
int_0^x frac{t^k}{left(t^n + aright)^m}:dt = frac{1}{n}a^{frac{k + 1}{n} - m} left[Bleft(m - frac{k + 1}{n}, frac{k + 1}{n}right) - Bleft(frac{1}{1 + ax^n}; m - frac{k + 1}{n}, frac{k + 1}{n} right)right]
end{equation}



Here we observe that for convergence:



begin{equation}
m - frac{k + 1}{n} gt 0,quad frac{k + 1}{n} gt 0,quad n neq 0
end{equation}



Note: when $x rightarrow infty$ we have:



begin{equation}
int_0^infty frac{t^k}{left(t^n + aright)^m}:dt = frac{1}{n}a^{frac{k + 1}{n} - m} Bleft(m - frac{k + 1}{n}, frac{k + 1}{n}right)
end{equation}





Update: Today I realised that we can use this result for another integral:



begin{equation}
int_0^infty frac{ln(t)}{left(t^n + 1right)^m}:dt
end{equation}



This is achieved through a simple use of Feynman's Trick. Here we consider the case when $x rightarrow infty$ and $a = 1$. We see that



begin{align}
frac{d}{dk}left[ int_0^infty frac{t^k}{left(t^n + 1right)^m}:dt right]&= frac{d}{dk}left[frac{1}{n}Bleft(m - frac{k + 1}{n}, frac{k + 1}{n} right)right] \
int_0^infty frac{t^k ln(t)}{left(t^n + 1right)^m}:dt &= frac{1}{n^2}Bleft(m - frac{k + 1}{n}, frac{k + 1}{n} right)left[psi^{(0)}left(frac{k + 1}{n}right) - psi^{(0)}left(m - frac{k + 1}{n}right) right]
end{align}



Thus,
begin{equation}
lim_{k rightarrow 0} int_0^infty frac{t^k ln(t)}{left(t^n + 1right)^m}:dt = lim_{k rightarrow 0}frac{1}{n^2}Bleft(m - frac{k + 1}{n}, frac{k + 1}{n} right)left[psi^{(0)}left(frac{k + 1}{n}right) - psi^{(0)}left(m - frac{k + 1}{n}right) right]
end{equation}



And finally:



begin{equation}
int_0^infty frac{ ln(t)}{left(t^n + 1right)^m}:dt = frac{1}{n^2}Bleft(m - frac{1}{n}, frac{1}{n} right)left[psi^{(0)}left(frac{1}{n}right) - psi^{(0)}left(m - frac{1}{n}right) right]
end{equation}



Note: In the case where $m = 1$ we arrive:



begin{align}
int_0^infty frac{ ln(t)}{left(t^n + 1right)^1}:dt &= frac{1}{n^2}Bleft(1 - frac{1}{n}, frac{1}{n} right)left[psi^{(0)}left(frac{1}{n}right) - psi^{(0)}left(1 - frac{1}{n}right) right] \
&= frac{1}{n^2} Gammaleft(frac{1}{n} right)Gammaleft(1 - frac{1}{n} right) cdot -picotleft(frac{pi}{n}right) \
&= frac{1}{n^2} frac{pi}{sinleft(frac{pi}{n}right)}cdot -picotleft(frac{pi}{n}right)
end{align}



Thus:



begin{equation}
int_0^infty frac{ ln(t)}{t^n + 1}:dt = -frac{pi^2}{n^2} operatorname{cosec}left(frac{pi}{n} right)cotleft(frac{pi}{n}right)
end{equation}










share|cite|improve this question











$endgroup$








  • 1




    $begingroup$
    damn bro that's a crazy integral
    $endgroup$
    – clathratus
    Dec 31 '18 at 6:07










  • $begingroup$
    It's more that I see it so often I wanted to finally get it out in a closed form so I could be done with solving it 10 different ways haha. Please if you have time look in detail. I do believe it's correct, but I need some wise eyes to have a look over.
    $endgroup$
    – DavidG
    Dec 31 '18 at 6:15










  • $begingroup$
    Would this work for $n=2$, $a=frac{1+b}{1-b}$ for some $|b|<1$, and some integer $mgeq1$, and $k=2K$ for integer $0leq Kleq m-1$? I need to know for $$F(m;b)=int_0^pi frac{mathrm dx}{(1+bcos x)^m}$$
    $endgroup$
    – clathratus
    Jan 17 at 18:03












  • $begingroup$
    @clathratus - Given that the integrand is discontinuous at $x = frac{pi}{2}$ - do you expect there to be an elementary primitive?. Regardless, if you changed the upper bound to $frac{pi}{2}$ then the half-tangent substitution should yield an elementary form.
    $endgroup$
    – DavidG
    Jan 18 at 9:36






  • 1




    $begingroup$
    Check out this: math.stackexchange.com/q/3077345/583016 It's also on our document. I have made a desmos page and it seems as if it works.
    $endgroup$
    – clathratus
    Jan 18 at 17:27
















9












$begingroup$


In working on integrals for the past couple of months, I've come across different cases of the following integral:



begin{equation}
Ileft(x,a,k,n,mright) = int_0^x frac{t^k}{left(t^n + aright)^m}:dt
end{equation}



Where $x,ain mathbb{R}^{+}$.



Here the method that I've taken is rather simple and I was curious as to other 'Real' Based methods could be employed with this integral? I also believe that with the conditions I've set on the parameters that it is convergent. If I'm able to expand those conditions, could you please advise.



Interested in special cases too!



The method I took:



First I wanted to bring the 'a' out the front:



begin{equation}
I(x,a,k,n,m) = int_0^x frac{t^k}{left(aleft[left(a^{-frac{1}{n}}tright)^n + 1right]right)^m}:dt = frac{1}{a^m} int_0^x frac{t^k}{left(left(a^{-frac{1}{n}}tright)^n + 1right)^m}:dt
end{equation}

Here let $u = a^{-frac{1}{n}}t$ Thus,



begin{equation}
I(x,a,k,n,m) = frac{1}{a^m} int_0^{a^{-frac{1}{n}}x} frac{left(a^{frac{1}{n}}uright)^k}{left(u^n + 1right)^m}a^{frac{1}{n}}:du = a^{frac{k + 1}{n} - m}int_0^{a^{-frac{1}{n}}x} frac{u^k}{left(u^n + 1right)^m}:du = a^{frac{k + 1}{n} - m}I(a^{-frac{1}{n}}x,1,k,n,m)
end{equation}



From here I will use $I$ in place of $I(x,a,k,n,m)$ for ease of typing. The next step is to make the substitution $w = u^n$ to yield:



begin{equation}
I = a^{frac{k + 1}{n} - m}int_0^{ax^n} frac{w^frac{k}{n}}{left(w + 1right)^m}frac{:dw}{nw^{frac{n - 1}{n}}} = frac{1}{n}a^{frac{k + 1}{n} - m}int_0^{ax^n} frac{w^{frac{k + 1}{n} - 1}}{left(w + 1right)^m}:dw
end{equation}



Here make the substitution $z = frac{1}{1 + w}$ to yield:



begin{align}
I &= frac{1}{n}a^{frac{k + 1}{n} - m}int_1^{frac{1}{1 + ax^n}} z^m left(frac{1 - z}{z}right)^{frac{k + 1}{n} - 1}left(-frac{1}{z^2}right) :dz = frac{1}{n}a^{frac{k + 1}{n} - m}int_{frac{1}{1 + ax^n}}^1 z^{m - frac{k + 1}{n} - 1}left(1 - zright)^{frac{k + 1}{n} - 1}:dz \
&= frac{1}{n}a^{frac{k + 1}{n} - m} left[int_0^1 z^{m - frac{k + 1}{n} - 1}left(1 - zright)^{frac{k + 1}{n} - 1}:dz - int_0^{frac{1}{1 + ax^n}} z^{m - frac{k + 1}{n} - 1}left(1 - zright)^{frac{k + 1}{n} - 1}:dz right] \
&= frac{1}{n}a^{frac{k + 1}{n} - m} left[Bleft(m - frac{k + 1}{n}, frac{k + 1}{n}right) - Bleft( frac{1}{1 + ax^n}; m - frac{k + 1}{n}, frac{k + 1}{n} right)right]
end{align}



Where $B(a,b)$ is the Beta Function and $B(x; a,b)$ is the Incomplete Beta Function.



And so, we arrive at:



begin{equation}
int_0^x frac{t^k}{left(t^n + aright)^m}:dt = frac{1}{n}a^{frac{k + 1}{n} - m} left[Bleft(m - frac{k + 1}{n}, frac{k + 1}{n}right) - Bleft(frac{1}{1 + ax^n}; m - frac{k + 1}{n}, frac{k + 1}{n} right)right]
end{equation}



Here we observe that for convergence:



begin{equation}
m - frac{k + 1}{n} gt 0,quad frac{k + 1}{n} gt 0,quad n neq 0
end{equation}



Note: when $x rightarrow infty$ we have:



begin{equation}
int_0^infty frac{t^k}{left(t^n + aright)^m}:dt = frac{1}{n}a^{frac{k + 1}{n} - m} Bleft(m - frac{k + 1}{n}, frac{k + 1}{n}right)
end{equation}





Update: Today I realised that we can use this result for another integral:



begin{equation}
int_0^infty frac{ln(t)}{left(t^n + 1right)^m}:dt
end{equation}



This is achieved through a simple use of Feynman's Trick. Here we consider the case when $x rightarrow infty$ and $a = 1$. We see that



begin{align}
frac{d}{dk}left[ int_0^infty frac{t^k}{left(t^n + 1right)^m}:dt right]&= frac{d}{dk}left[frac{1}{n}Bleft(m - frac{k + 1}{n}, frac{k + 1}{n} right)right] \
int_0^infty frac{t^k ln(t)}{left(t^n + 1right)^m}:dt &= frac{1}{n^2}Bleft(m - frac{k + 1}{n}, frac{k + 1}{n} right)left[psi^{(0)}left(frac{k + 1}{n}right) - psi^{(0)}left(m - frac{k + 1}{n}right) right]
end{align}



Thus,
begin{equation}
lim_{k rightarrow 0} int_0^infty frac{t^k ln(t)}{left(t^n + 1right)^m}:dt = lim_{k rightarrow 0}frac{1}{n^2}Bleft(m - frac{k + 1}{n}, frac{k + 1}{n} right)left[psi^{(0)}left(frac{k + 1}{n}right) - psi^{(0)}left(m - frac{k + 1}{n}right) right]
end{equation}



And finally:



begin{equation}
int_0^infty frac{ ln(t)}{left(t^n + 1right)^m}:dt = frac{1}{n^2}Bleft(m - frac{1}{n}, frac{1}{n} right)left[psi^{(0)}left(frac{1}{n}right) - psi^{(0)}left(m - frac{1}{n}right) right]
end{equation}



Note: In the case where $m = 1$ we arrive:



begin{align}
int_0^infty frac{ ln(t)}{left(t^n + 1right)^1}:dt &= frac{1}{n^2}Bleft(1 - frac{1}{n}, frac{1}{n} right)left[psi^{(0)}left(frac{1}{n}right) - psi^{(0)}left(1 - frac{1}{n}right) right] \
&= frac{1}{n^2} Gammaleft(frac{1}{n} right)Gammaleft(1 - frac{1}{n} right) cdot -picotleft(frac{pi}{n}right) \
&= frac{1}{n^2} frac{pi}{sinleft(frac{pi}{n}right)}cdot -picotleft(frac{pi}{n}right)
end{align}



Thus:



begin{equation}
int_0^infty frac{ ln(t)}{t^n + 1}:dt = -frac{pi^2}{n^2} operatorname{cosec}left(frac{pi}{n} right)cotleft(frac{pi}{n}right)
end{equation}










share|cite|improve this question











$endgroup$








  • 1




    $begingroup$
    damn bro that's a crazy integral
    $endgroup$
    – clathratus
    Dec 31 '18 at 6:07










  • $begingroup$
    It's more that I see it so often I wanted to finally get it out in a closed form so I could be done with solving it 10 different ways haha. Please if you have time look in detail. I do believe it's correct, but I need some wise eyes to have a look over.
    $endgroup$
    – DavidG
    Dec 31 '18 at 6:15










  • $begingroup$
    Would this work for $n=2$, $a=frac{1+b}{1-b}$ for some $|b|<1$, and some integer $mgeq1$, and $k=2K$ for integer $0leq Kleq m-1$? I need to know for $$F(m;b)=int_0^pi frac{mathrm dx}{(1+bcos x)^m}$$
    $endgroup$
    – clathratus
    Jan 17 at 18:03












  • $begingroup$
    @clathratus - Given that the integrand is discontinuous at $x = frac{pi}{2}$ - do you expect there to be an elementary primitive?. Regardless, if you changed the upper bound to $frac{pi}{2}$ then the half-tangent substitution should yield an elementary form.
    $endgroup$
    – DavidG
    Jan 18 at 9:36






  • 1




    $begingroup$
    Check out this: math.stackexchange.com/q/3077345/583016 It's also on our document. I have made a desmos page and it seems as if it works.
    $endgroup$
    – clathratus
    Jan 18 at 17:27














9












9








9


4



$begingroup$


In working on integrals for the past couple of months, I've come across different cases of the following integral:



begin{equation}
Ileft(x,a,k,n,mright) = int_0^x frac{t^k}{left(t^n + aright)^m}:dt
end{equation}



Where $x,ain mathbb{R}^{+}$.



Here the method that I've taken is rather simple and I was curious as to other 'Real' Based methods could be employed with this integral? I also believe that with the conditions I've set on the parameters that it is convergent. If I'm able to expand those conditions, could you please advise.



Interested in special cases too!



The method I took:



First I wanted to bring the 'a' out the front:



begin{equation}
I(x,a,k,n,m) = int_0^x frac{t^k}{left(aleft[left(a^{-frac{1}{n}}tright)^n + 1right]right)^m}:dt = frac{1}{a^m} int_0^x frac{t^k}{left(left(a^{-frac{1}{n}}tright)^n + 1right)^m}:dt
end{equation}

Here let $u = a^{-frac{1}{n}}t$ Thus,



begin{equation}
I(x,a,k,n,m) = frac{1}{a^m} int_0^{a^{-frac{1}{n}}x} frac{left(a^{frac{1}{n}}uright)^k}{left(u^n + 1right)^m}a^{frac{1}{n}}:du = a^{frac{k + 1}{n} - m}int_0^{a^{-frac{1}{n}}x} frac{u^k}{left(u^n + 1right)^m}:du = a^{frac{k + 1}{n} - m}I(a^{-frac{1}{n}}x,1,k,n,m)
end{equation}



From here I will use $I$ in place of $I(x,a,k,n,m)$ for ease of typing. The next step is to make the substitution $w = u^n$ to yield:



begin{equation}
I = a^{frac{k + 1}{n} - m}int_0^{ax^n} frac{w^frac{k}{n}}{left(w + 1right)^m}frac{:dw}{nw^{frac{n - 1}{n}}} = frac{1}{n}a^{frac{k + 1}{n} - m}int_0^{ax^n} frac{w^{frac{k + 1}{n} - 1}}{left(w + 1right)^m}:dw
end{equation}



Here make the substitution $z = frac{1}{1 + w}$ to yield:



begin{align}
I &= frac{1}{n}a^{frac{k + 1}{n} - m}int_1^{frac{1}{1 + ax^n}} z^m left(frac{1 - z}{z}right)^{frac{k + 1}{n} - 1}left(-frac{1}{z^2}right) :dz = frac{1}{n}a^{frac{k + 1}{n} - m}int_{frac{1}{1 + ax^n}}^1 z^{m - frac{k + 1}{n} - 1}left(1 - zright)^{frac{k + 1}{n} - 1}:dz \
&= frac{1}{n}a^{frac{k + 1}{n} - m} left[int_0^1 z^{m - frac{k + 1}{n} - 1}left(1 - zright)^{frac{k + 1}{n} - 1}:dz - int_0^{frac{1}{1 + ax^n}} z^{m - frac{k + 1}{n} - 1}left(1 - zright)^{frac{k + 1}{n} - 1}:dz right] \
&= frac{1}{n}a^{frac{k + 1}{n} - m} left[Bleft(m - frac{k + 1}{n}, frac{k + 1}{n}right) - Bleft( frac{1}{1 + ax^n}; m - frac{k + 1}{n}, frac{k + 1}{n} right)right]
end{align}



Where $B(a,b)$ is the Beta Function and $B(x; a,b)$ is the Incomplete Beta Function.



And so, we arrive at:



begin{equation}
int_0^x frac{t^k}{left(t^n + aright)^m}:dt = frac{1}{n}a^{frac{k + 1}{n} - m} left[Bleft(m - frac{k + 1}{n}, frac{k + 1}{n}right) - Bleft(frac{1}{1 + ax^n}; m - frac{k + 1}{n}, frac{k + 1}{n} right)right]
end{equation}



Here we observe that for convergence:



begin{equation}
m - frac{k + 1}{n} gt 0,quad frac{k + 1}{n} gt 0,quad n neq 0
end{equation}



Note: when $x rightarrow infty$ we have:



begin{equation}
int_0^infty frac{t^k}{left(t^n + aright)^m}:dt = frac{1}{n}a^{frac{k + 1}{n} - m} Bleft(m - frac{k + 1}{n}, frac{k + 1}{n}right)
end{equation}





Update: Today I realised that we can use this result for another integral:



begin{equation}
int_0^infty frac{ln(t)}{left(t^n + 1right)^m}:dt
end{equation}



This is achieved through a simple use of Feynman's Trick. Here we consider the case when $x rightarrow infty$ and $a = 1$. We see that



begin{align}
frac{d}{dk}left[ int_0^infty frac{t^k}{left(t^n + 1right)^m}:dt right]&= frac{d}{dk}left[frac{1}{n}Bleft(m - frac{k + 1}{n}, frac{k + 1}{n} right)right] \
int_0^infty frac{t^k ln(t)}{left(t^n + 1right)^m}:dt &= frac{1}{n^2}Bleft(m - frac{k + 1}{n}, frac{k + 1}{n} right)left[psi^{(0)}left(frac{k + 1}{n}right) - psi^{(0)}left(m - frac{k + 1}{n}right) right]
end{align}



Thus,
begin{equation}
lim_{k rightarrow 0} int_0^infty frac{t^k ln(t)}{left(t^n + 1right)^m}:dt = lim_{k rightarrow 0}frac{1}{n^2}Bleft(m - frac{k + 1}{n}, frac{k + 1}{n} right)left[psi^{(0)}left(frac{k + 1}{n}right) - psi^{(0)}left(m - frac{k + 1}{n}right) right]
end{equation}



And finally:



begin{equation}
int_0^infty frac{ ln(t)}{left(t^n + 1right)^m}:dt = frac{1}{n^2}Bleft(m - frac{1}{n}, frac{1}{n} right)left[psi^{(0)}left(frac{1}{n}right) - psi^{(0)}left(m - frac{1}{n}right) right]
end{equation}



Note: In the case where $m = 1$ we arrive:



begin{align}
int_0^infty frac{ ln(t)}{left(t^n + 1right)^1}:dt &= frac{1}{n^2}Bleft(1 - frac{1}{n}, frac{1}{n} right)left[psi^{(0)}left(frac{1}{n}right) - psi^{(0)}left(1 - frac{1}{n}right) right] \
&= frac{1}{n^2} Gammaleft(frac{1}{n} right)Gammaleft(1 - frac{1}{n} right) cdot -picotleft(frac{pi}{n}right) \
&= frac{1}{n^2} frac{pi}{sinleft(frac{pi}{n}right)}cdot -picotleft(frac{pi}{n}right)
end{align}



Thus:



begin{equation}
int_0^infty frac{ ln(t)}{t^n + 1}:dt = -frac{pi^2}{n^2} operatorname{cosec}left(frac{pi}{n} right)cotleft(frac{pi}{n}right)
end{equation}










share|cite|improve this question











$endgroup$




In working on integrals for the past couple of months, I've come across different cases of the following integral:



begin{equation}
Ileft(x,a,k,n,mright) = int_0^x frac{t^k}{left(t^n + aright)^m}:dt
end{equation}



Where $x,ain mathbb{R}^{+}$.



Here the method that I've taken is rather simple and I was curious as to other 'Real' Based methods could be employed with this integral? I also believe that with the conditions I've set on the parameters that it is convergent. If I'm able to expand those conditions, could you please advise.



Interested in special cases too!



The method I took:



First I wanted to bring the 'a' out the front:



begin{equation}
I(x,a,k,n,m) = int_0^x frac{t^k}{left(aleft[left(a^{-frac{1}{n}}tright)^n + 1right]right)^m}:dt = frac{1}{a^m} int_0^x frac{t^k}{left(left(a^{-frac{1}{n}}tright)^n + 1right)^m}:dt
end{equation}

Here let $u = a^{-frac{1}{n}}t$ Thus,



begin{equation}
I(x,a,k,n,m) = frac{1}{a^m} int_0^{a^{-frac{1}{n}}x} frac{left(a^{frac{1}{n}}uright)^k}{left(u^n + 1right)^m}a^{frac{1}{n}}:du = a^{frac{k + 1}{n} - m}int_0^{a^{-frac{1}{n}}x} frac{u^k}{left(u^n + 1right)^m}:du = a^{frac{k + 1}{n} - m}I(a^{-frac{1}{n}}x,1,k,n,m)
end{equation}



From here I will use $I$ in place of $I(x,a,k,n,m)$ for ease of typing. The next step is to make the substitution $w = u^n$ to yield:



begin{equation}
I = a^{frac{k + 1}{n} - m}int_0^{ax^n} frac{w^frac{k}{n}}{left(w + 1right)^m}frac{:dw}{nw^{frac{n - 1}{n}}} = frac{1}{n}a^{frac{k + 1}{n} - m}int_0^{ax^n} frac{w^{frac{k + 1}{n} - 1}}{left(w + 1right)^m}:dw
end{equation}



Here make the substitution $z = frac{1}{1 + w}$ to yield:



begin{align}
I &= frac{1}{n}a^{frac{k + 1}{n} - m}int_1^{frac{1}{1 + ax^n}} z^m left(frac{1 - z}{z}right)^{frac{k + 1}{n} - 1}left(-frac{1}{z^2}right) :dz = frac{1}{n}a^{frac{k + 1}{n} - m}int_{frac{1}{1 + ax^n}}^1 z^{m - frac{k + 1}{n} - 1}left(1 - zright)^{frac{k + 1}{n} - 1}:dz \
&= frac{1}{n}a^{frac{k + 1}{n} - m} left[int_0^1 z^{m - frac{k + 1}{n} - 1}left(1 - zright)^{frac{k + 1}{n} - 1}:dz - int_0^{frac{1}{1 + ax^n}} z^{m - frac{k + 1}{n} - 1}left(1 - zright)^{frac{k + 1}{n} - 1}:dz right] \
&= frac{1}{n}a^{frac{k + 1}{n} - m} left[Bleft(m - frac{k + 1}{n}, frac{k + 1}{n}right) - Bleft( frac{1}{1 + ax^n}; m - frac{k + 1}{n}, frac{k + 1}{n} right)right]
end{align}



Where $B(a,b)$ is the Beta Function and $B(x; a,b)$ is the Incomplete Beta Function.



And so, we arrive at:



begin{equation}
int_0^x frac{t^k}{left(t^n + aright)^m}:dt = frac{1}{n}a^{frac{k + 1}{n} - m} left[Bleft(m - frac{k + 1}{n}, frac{k + 1}{n}right) - Bleft(frac{1}{1 + ax^n}; m - frac{k + 1}{n}, frac{k + 1}{n} right)right]
end{equation}



Here we observe that for convergence:



begin{equation}
m - frac{k + 1}{n} gt 0,quad frac{k + 1}{n} gt 0,quad n neq 0
end{equation}



Note: when $x rightarrow infty$ we have:



begin{equation}
int_0^infty frac{t^k}{left(t^n + aright)^m}:dt = frac{1}{n}a^{frac{k + 1}{n} - m} Bleft(m - frac{k + 1}{n}, frac{k + 1}{n}right)
end{equation}





Update: Today I realised that we can use this result for another integral:



begin{equation}
int_0^infty frac{ln(t)}{left(t^n + 1right)^m}:dt
end{equation}



This is achieved through a simple use of Feynman's Trick. Here we consider the case when $x rightarrow infty$ and $a = 1$. We see that



begin{align}
frac{d}{dk}left[ int_0^infty frac{t^k}{left(t^n + 1right)^m}:dt right]&= frac{d}{dk}left[frac{1}{n}Bleft(m - frac{k + 1}{n}, frac{k + 1}{n} right)right] \
int_0^infty frac{t^k ln(t)}{left(t^n + 1right)^m}:dt &= frac{1}{n^2}Bleft(m - frac{k + 1}{n}, frac{k + 1}{n} right)left[psi^{(0)}left(frac{k + 1}{n}right) - psi^{(0)}left(m - frac{k + 1}{n}right) right]
end{align}



Thus,
begin{equation}
lim_{k rightarrow 0} int_0^infty frac{t^k ln(t)}{left(t^n + 1right)^m}:dt = lim_{k rightarrow 0}frac{1}{n^2}Bleft(m - frac{k + 1}{n}, frac{k + 1}{n} right)left[psi^{(0)}left(frac{k + 1}{n}right) - psi^{(0)}left(m - frac{k + 1}{n}right) right]
end{equation}



And finally:



begin{equation}
int_0^infty frac{ ln(t)}{left(t^n + 1right)^m}:dt = frac{1}{n^2}Bleft(m - frac{1}{n}, frac{1}{n} right)left[psi^{(0)}left(frac{1}{n}right) - psi^{(0)}left(m - frac{1}{n}right) right]
end{equation}



Note: In the case where $m = 1$ we arrive:



begin{align}
int_0^infty frac{ ln(t)}{left(t^n + 1right)^1}:dt &= frac{1}{n^2}Bleft(1 - frac{1}{n}, frac{1}{n} right)left[psi^{(0)}left(frac{1}{n}right) - psi^{(0)}left(1 - frac{1}{n}right) right] \
&= frac{1}{n^2} Gammaleft(frac{1}{n} right)Gammaleft(1 - frac{1}{n} right) cdot -picotleft(frac{pi}{n}right) \
&= frac{1}{n^2} frac{pi}{sinleft(frac{pi}{n}right)}cdot -picotleft(frac{pi}{n}right)
end{align}



Thus:



begin{equation}
int_0^infty frac{ ln(t)}{t^n + 1}:dt = -frac{pi^2}{n^2} operatorname{cosec}left(frac{pi}{n} right)cotleft(frac{pi}{n}right)
end{equation}







real-analysis integration definite-integrals beta-function






share|cite|improve this question















share|cite|improve this question













share|cite|improve this question




share|cite|improve this question








edited Mar 13 at 3:46









NickD

1,1921512




1,1921512










asked Dec 30 '18 at 23:58









DavidGDavidG

1




1








  • 1




    $begingroup$
    damn bro that's a crazy integral
    $endgroup$
    – clathratus
    Dec 31 '18 at 6:07










  • $begingroup$
    It's more that I see it so often I wanted to finally get it out in a closed form so I could be done with solving it 10 different ways haha. Please if you have time look in detail. I do believe it's correct, but I need some wise eyes to have a look over.
    $endgroup$
    – DavidG
    Dec 31 '18 at 6:15










  • $begingroup$
    Would this work for $n=2$, $a=frac{1+b}{1-b}$ for some $|b|<1$, and some integer $mgeq1$, and $k=2K$ for integer $0leq Kleq m-1$? I need to know for $$F(m;b)=int_0^pi frac{mathrm dx}{(1+bcos x)^m}$$
    $endgroup$
    – clathratus
    Jan 17 at 18:03












  • $begingroup$
    @clathratus - Given that the integrand is discontinuous at $x = frac{pi}{2}$ - do you expect there to be an elementary primitive?. Regardless, if you changed the upper bound to $frac{pi}{2}$ then the half-tangent substitution should yield an elementary form.
    $endgroup$
    – DavidG
    Jan 18 at 9:36






  • 1




    $begingroup$
    Check out this: math.stackexchange.com/q/3077345/583016 It's also on our document. I have made a desmos page and it seems as if it works.
    $endgroup$
    – clathratus
    Jan 18 at 17:27














  • 1




    $begingroup$
    damn bro that's a crazy integral
    $endgroup$
    – clathratus
    Dec 31 '18 at 6:07










  • $begingroup$
    It's more that I see it so often I wanted to finally get it out in a closed form so I could be done with solving it 10 different ways haha. Please if you have time look in detail. I do believe it's correct, but I need some wise eyes to have a look over.
    $endgroup$
    – DavidG
    Dec 31 '18 at 6:15










  • $begingroup$
    Would this work for $n=2$, $a=frac{1+b}{1-b}$ for some $|b|<1$, and some integer $mgeq1$, and $k=2K$ for integer $0leq Kleq m-1$? I need to know for $$F(m;b)=int_0^pi frac{mathrm dx}{(1+bcos x)^m}$$
    $endgroup$
    – clathratus
    Jan 17 at 18:03












  • $begingroup$
    @clathratus - Given that the integrand is discontinuous at $x = frac{pi}{2}$ - do you expect there to be an elementary primitive?. Regardless, if you changed the upper bound to $frac{pi}{2}$ then the half-tangent substitution should yield an elementary form.
    $endgroup$
    – DavidG
    Jan 18 at 9:36






  • 1




    $begingroup$
    Check out this: math.stackexchange.com/q/3077345/583016 It's also on our document. I have made a desmos page and it seems as if it works.
    $endgroup$
    – clathratus
    Jan 18 at 17:27








1




1




$begingroup$
damn bro that's a crazy integral
$endgroup$
– clathratus
Dec 31 '18 at 6:07




$begingroup$
damn bro that's a crazy integral
$endgroup$
– clathratus
Dec 31 '18 at 6:07












$begingroup$
It's more that I see it so often I wanted to finally get it out in a closed form so I could be done with solving it 10 different ways haha. Please if you have time look in detail. I do believe it's correct, but I need some wise eyes to have a look over.
$endgroup$
– DavidG
Dec 31 '18 at 6:15




$begingroup$
It's more that I see it so often I wanted to finally get it out in a closed form so I could be done with solving it 10 different ways haha. Please if you have time look in detail. I do believe it's correct, but I need some wise eyes to have a look over.
$endgroup$
– DavidG
Dec 31 '18 at 6:15












$begingroup$
Would this work for $n=2$, $a=frac{1+b}{1-b}$ for some $|b|<1$, and some integer $mgeq1$, and $k=2K$ for integer $0leq Kleq m-1$? I need to know for $$F(m;b)=int_0^pi frac{mathrm dx}{(1+bcos x)^m}$$
$endgroup$
– clathratus
Jan 17 at 18:03






$begingroup$
Would this work for $n=2$, $a=frac{1+b}{1-b}$ for some $|b|<1$, and some integer $mgeq1$, and $k=2K$ for integer $0leq Kleq m-1$? I need to know for $$F(m;b)=int_0^pi frac{mathrm dx}{(1+bcos x)^m}$$
$endgroup$
– clathratus
Jan 17 at 18:03














$begingroup$
@clathratus - Given that the integrand is discontinuous at $x = frac{pi}{2}$ - do you expect there to be an elementary primitive?. Regardless, if you changed the upper bound to $frac{pi}{2}$ then the half-tangent substitution should yield an elementary form.
$endgroup$
– DavidG
Jan 18 at 9:36




$begingroup$
@clathratus - Given that the integrand is discontinuous at $x = frac{pi}{2}$ - do you expect there to be an elementary primitive?. Regardless, if you changed the upper bound to $frac{pi}{2}$ then the half-tangent substitution should yield an elementary form.
$endgroup$
– DavidG
Jan 18 at 9:36




1




1




$begingroup$
Check out this: math.stackexchange.com/q/3077345/583016 It's also on our document. I have made a desmos page and it seems as if it works.
$endgroup$
– clathratus
Jan 18 at 17:27




$begingroup$
Check out this: math.stackexchange.com/q/3077345/583016 It's also on our document. I have made a desmos page and it seems as if it works.
$endgroup$
– clathratus
Jan 18 at 17:27










1 Answer
1






active

oldest

votes


















2












$begingroup$

NOT A SOLUTION:



I've found some special cases on this site that I will list (this will evolve as I find more special (but generalised) cases:




  1. Closed form for $ int_0^infty {frac{{{x^n}}}{{1 + {x^m}}}dx }$


  2. Evaluate the integral $ int _0^{+infty} frac{x^m}{(a+bx^n)^p}$






share|cite|improve this answer











$endgroup$













    Your Answer





    StackExchange.ifUsing("editor", function () {
    return StackExchange.using("mathjaxEditing", function () {
    StackExchange.MarkdownEditor.creationCallbacks.add(function (editor, postfix) {
    StackExchange.mathjaxEditing.prepareWmdForMathJax(editor, postfix, [["$", "$"], ["\\(","\\)"]]);
    });
    });
    }, "mathjax-editing");

    StackExchange.ready(function() {
    var channelOptions = {
    tags: "".split(" "),
    id: "69"
    };
    initTagRenderer("".split(" "), "".split(" "), channelOptions);

    StackExchange.using("externalEditor", function() {
    // Have to fire editor after snippets, if snippets enabled
    if (StackExchange.settings.snippets.snippetsEnabled) {
    StackExchange.using("snippets", function() {
    createEditor();
    });
    }
    else {
    createEditor();
    }
    });

    function createEditor() {
    StackExchange.prepareEditor({
    heartbeatType: 'answer',
    autoActivateHeartbeat: false,
    convertImagesToLinks: true,
    noModals: true,
    showLowRepImageUploadWarning: true,
    reputationToPostImages: 10,
    bindNavPrevention: true,
    postfix: "",
    imageUploader: {
    brandingHtml: "Powered by u003ca class="icon-imgur-white" href="https://imgur.com/"u003eu003c/au003e",
    contentPolicyHtml: "User contributions licensed under u003ca href="https://creativecommons.org/licenses/by-sa/3.0/"u003ecc by-sa 3.0 with attribution requiredu003c/au003e u003ca href="https://stackoverflow.com/legal/content-policy"u003e(content policy)u003c/au003e",
    allowUrls: true
    },
    noCode: true, onDemand: true,
    discardSelector: ".discard-answer"
    ,immediatelyShowMarkdownHelp:true
    });


    }
    });














    draft saved

    draft discarded


















    StackExchange.ready(
    function () {
    StackExchange.openid.initPostLogin('.new-post-login', 'https%3a%2f%2fmath.stackexchange.com%2fquestions%2f3057298%2fsolving-used-real-based-methods-int-0x-fractk-lefttn-a-rightm-d%23new-answer', 'question_page');
    }
    );

    Post as a guest















    Required, but never shown

























    1 Answer
    1






    active

    oldest

    votes








    1 Answer
    1






    active

    oldest

    votes









    active

    oldest

    votes






    active

    oldest

    votes









    2












    $begingroup$

    NOT A SOLUTION:



    I've found some special cases on this site that I will list (this will evolve as I find more special (but generalised) cases:




    1. Closed form for $ int_0^infty {frac{{{x^n}}}{{1 + {x^m}}}dx }$


    2. Evaluate the integral $ int _0^{+infty} frac{x^m}{(a+bx^n)^p}$






    share|cite|improve this answer











    $endgroup$


















      2












      $begingroup$

      NOT A SOLUTION:



      I've found some special cases on this site that I will list (this will evolve as I find more special (but generalised) cases:




      1. Closed form for $ int_0^infty {frac{{{x^n}}}{{1 + {x^m}}}dx }$


      2. Evaluate the integral $ int _0^{+infty} frac{x^m}{(a+bx^n)^p}$






      share|cite|improve this answer











      $endgroup$
















        2












        2








        2





        $begingroup$

        NOT A SOLUTION:



        I've found some special cases on this site that I will list (this will evolve as I find more special (but generalised) cases:




        1. Closed form for $ int_0^infty {frac{{{x^n}}}{{1 + {x^m}}}dx }$


        2. Evaluate the integral $ int _0^{+infty} frac{x^m}{(a+bx^n)^p}$






        share|cite|improve this answer











        $endgroup$



        NOT A SOLUTION:



        I've found some special cases on this site that I will list (this will evolve as I find more special (but generalised) cases:




        1. Closed form for $ int_0^infty {frac{{{x^n}}}{{1 + {x^m}}}dx }$


        2. Evaluate the integral $ int _0^{+infty} frac{x^m}{(a+bx^n)^p}$







        share|cite|improve this answer














        share|cite|improve this answer



        share|cite|improve this answer








        edited Jan 9 at 0:19

























        answered Jan 2 at 0:29









        DavidGDavidG

        1




        1






























            draft saved

            draft discarded




















































            Thanks for contributing an answer to Mathematics Stack Exchange!


            • Please be sure to answer the question. Provide details and share your research!

            But avoid



            • Asking for help, clarification, or responding to other answers.

            • Making statements based on opinion; back them up with references or personal experience.


            Use MathJax to format equations. MathJax reference.


            To learn more, see our tips on writing great answers.




            draft saved


            draft discarded














            StackExchange.ready(
            function () {
            StackExchange.openid.initPostLogin('.new-post-login', 'https%3a%2f%2fmath.stackexchange.com%2fquestions%2f3057298%2fsolving-used-real-based-methods-int-0x-fractk-lefttn-a-rightm-d%23new-answer', 'question_page');
            }
            );

            Post as a guest















            Required, but never shown





















































            Required, but never shown














            Required, but never shown












            Required, but never shown







            Required, but never shown

































            Required, but never shown














            Required, but never shown












            Required, but never shown







            Required, but never shown







            Popular posts from this blog

            How do I know what Microsoft account the skydrive app is syncing to?

            When does type information flow backwards in C++?

            Grease: Live!